Modul English Club-Upt Pusat Bahasa Stieken

Anda mungkin juga menyukai

Anda di halaman 1dari 54

UPT.

PUSAT BAHASA
STIE KESUMA NEGARA BLITAR
JL. MASTRIP NO. 59 KOTA BLITAR
TELP. (0342) 802330

Center for International Language Development | 1

'LSDNDLXQWXNNDODQJDQVHQGLUL)RULQWHUQDOXVH
TEST OF ENGLISH AS A FOREIGN LANGUAGE

TOEFL

TOEFL merupakan singkatan dari Test of English as a Foreign Language. TOEFL adalah
standardisasi kemampuan bahasa Inggris seseorang secara tertulis yang meliputi empat
aspek penguasaan: Listening, Writing, Reading, dan Speaking. Namun, untuk model Paper
Based Test hanya terdapat tiga aspek yang diujikan: Listening Comprehension, Structure
and Written Expression, dan Reading Comprehension.

Tujuan Test TOEFL

TOEFL memliki tujuan khusus yakni untuk tujuan pendidikan, penelitian atau yang
berhubungan dengan kegiatan akademis di luar negeri, ataupun di Indonesia. Untuk jenjang
pascasarjana (S2) biasanya memiliki syarat nilai minimal 500 sedangkan untuk jenjang
sarjana (S1) memiliki syarat nilai minimal 450.

Disamping itu, ada kalanya TOEFL dipakai dalam bidang umum seperti pekerjaan, kenaikan
pangkat atau tugas kerja. Banyak perusahaan yang memasang standar Bahasa Inggris
karyawannya dengan melihat nilai TOEFL. Umumnya, nilai TOEFL minimal adalah 450-500
untuk kenaikan pangkat standar.

Struktur TOEFL

Ada tiga bagian tes TOEFL yang harus dikerjakan oleh peserta. Bagian pertama adalah soal-
soal yang mengukur kemampuan Listening Comprehension (50 soal), Structure & Written
Expression (40 soal), dan Reading Comprehension (50 soal). Keseluruhan soal dibuat dalam
bentuk pilihan berganda. Keseluruhan tes berlangsung kurang lebih 120 menit, untuk Paper
Based Test (PBT), dan kurang lebih 240 menit untuk Internet Based Test (IBT).

Klasifikasi UMUM Nilai TOEFL

Secara umum kita mengenal tiga level penguasaan bahasa asing, yaitu Tingkat Dasar
(Elementary), Tingkat Menengah (Intermediate), dan Tingkat Mahir (Advanced). Untuk
skor TOEFL, para ahli bahasa biasanya mengelompokkan skor ini kedalam empat level
berikut (Carson, et al., 1990):

· Tingkat Dasar (Elementary) : 310 s.d. 420


· Tingkat Menengah Bawah (Low Intermediate) : 420 s.d. 480
· Tingkat Menengah Atas (High Intermediate) : 480 s.d. 520
· Tingkat Mahir (Advanced) : 525 s.d 677

Penilaian TOEFL

Sistem penilaian TOEFL menggunakan konversi dari setiap jawaban yang benar. Nilai
TOEFL tertinggi yang bisa dicapai seseorang adalah 677.

2
UNIT I

LISTENING COMPREHENSION
Waktu = 35 menit (termasuk pembacaan petunjuk pelaksanaan
untuk setiap bagian) Tes ini memberikan kesempatan untuk
menunjukkan kemampuan memahami percakapan dan
pembicaraan dalam bahasa Inggris. Listening Comprehension
Test terdiri dari 3 (tiga) bagian:

Part A : SHORT DIALOGUE (DIALOG PENDEK)

Pada bagian ini, terdapat percakapan singkat antara dua orang.


Setelah percakapan, pertanyaan yang berhubungan dengan
percakapan tersebut akan diberikan. Baik percakapan dan pertanyaan tidak akan diulangi.

Part B : LONG CONVERSATION (PERCAKAPAN PANJANG)

Pada bagian ini, terdapat percakapan dengan topik santai antara beberapa orang dengan
durasi sekitar 60-90 detik. Setelah percakapan panjang selesai akan diikuti oleh sejumlah
pertanyaan pilihan ganda yang kesemuanya merujuk pada percakapan panjang tersebut.

Part C : TALKS (PEMBICARAAN)

Pada bagian ini, terdapat beberapa pembicaraan dengan durasi sekitar 60-90 detik tentang
kehidupan sekolah atau perkuliahan, masing-masing diikuti dengan pertanyaan pilihan
ganda.

1.1. DIALOG PENDEK

SKILL 1: FOCUS ON THE LAST LINE AND RESTATE


1
Seringkali jawaban yang benar untuk sebuah pertanyaan dalam bagian dialog pendek adalah
jawaban yang berisi penyajian kembali ide-ide dalam baris terakhir dari dialog. Jika anda
tidak sempat menangkap maksud speaker pertama, maka jangan khawatir karena biasanya
informasi terkait jawaban berada pada speaker kedua. Kemudian, cobalah untuk
menyatakannya kembali ke dalam sebuah kalimat lain yang memiliki kesamaan arti

Example:

On the recording, you hear:


(Woman) Steve, is something the matter? You don’t look very good
(Man) Oh, I’m feeling a little sick today
(Narrator) What does the man mean?
In your test book you read:

(A) He’s very good looking (C) He looks worse than he feels
(B) He’s a bit ill (D) His feet are a little thick

3
Garis bawahi ide pokok dari dialog singkat dibawah.

1. (woman) What do you like about your 2. (man) Mark said some really nice
new house? things to me
(man) It’s very close to a park (woman) He’s very grateful for what
(woman)What does the man mean? you did
(narrator) What does the woman say
(A) The house is closed up now about Mark?
(B) He parks his car close to his house
(C) His home is near a park (A) He did a great job
(D) He doesn’t really like his new house (B) He bought a crate full of fruit
(C) He made a great fool of himself
(D) He’s thankful

TOEFL EXERCISE 1:
In this exercise, listen carefully to each short dialogue and
question on the recording, and then choose the best answer to the
question. You should look for synonyms for key words in the last
line.

Now begin the recording at TOEFL exercise 1.

1. (A) He’d really like to have 4. (A) She is completely prepared for the exam.
(B) The man should have
something to eat.
studied harder.
(B) Because he’s weak, he can’t
(C) Night is the best time to study for exams.
eat.
(D) The exam was postponed.
(C) It’s been weeks since he’s had
anything to eat. 5. (A) Please give me your hand.
(D) He hasn’t felt like eating for (B) My hand is stuck under the packages.
weeks. (C) Please remove your
2. (A) He used his fortune to pay his hands from those
fees. packages.
(B) He was lucky to receive a (D) Would you help me carry
grant for his studies. these packages?
(C) He is a scholar at a college with
low fees.
(D)He paid to get scholarship.
3. (A) Mr. David pointedly asked the
president about the committee.
(B) The president to Mr. David’s
head.
(C) Mr. David became head of the
new commission.
(D) Mr. David was committed to the

2
president’s appointment.

Skill 2: NEGATIVES (Pernyataan Negatif)

Pernyataan negatif seringkali digunakan dalam dialog singkat. Respon yang paling umum
respon untuk pernyataan negatif adalah pernyataan positif yang mengandung sebuah kata
dengan arti yang berlawanan.
4
Example:

On the recording, you hear:

(woman) Did you get a lot of work done at the library today?
(man) I couldn’t. It wasn’t very quiet there
(narrator) What does the man mean?

In your test book, you read:

(A) The library was noisy


(B) He got a lot done
(C) He couldn’t quite get to the library today
(D) The library’s a good place to work because it’s quite

Jawaban yang benar adalah jawaban (A). Jika tidak tenang di perpustakaan, berarti
berisik. Perhatikan bahwa jawaban yang benar menggunakan berisik, kebalikan dari
tenang.

Garisbawahi ungkapan negatif di baris terakhir dari setiap dialog singkat.


Ingat bahwa jawaban yang paling tepat adalah yang menggunakan makna
berlawanan.

1. (woman) You made so many mistakes 2. (Man)Is there a lot of soup? I’m kind of
in this homework hungry
(man) I wasn’t very careful (woman) Sorry, there’s not a lot
(narrator) What does the man mean? (narrator) What does the woman
mean?
(A) He was rather careless
(B) He does not care about mistakes (A) There’s not very much soap
(C) He took care of the work at home (B) She doesn’t like soap
(D) He did not carry the work home (C) There’s only a little soup
(D) The man should not be hungry

5
TOEFL EXERCISE 2: In this exercise, listen carefully to each
short dialogue and question on the recording, and then choose the
best answer to the question. You should be particularly careful of
negative expressions.

1. (A) He’d really like to have something meeting begins.


to eat. (B) The meeting will not start after 10:oo.
(B) Because he’s weak, he can’t eat. (C) (C) The meeting is not at 10:00.
(C) it’s been weeks since he’s had
(D) He isn’t supposed to attend the meeting.
anything to eat.
(D) He hasn’t felt like eating for weeks. 4. (A) Use a different type of viewing device.
2. (A) The road that the horses (B) Stop rising the sign.
took was long and hard. (C) Stand up.
(B) It was hard to find the hidden (D) Sit down.
houses. 5. (A) Bill had never really been sick.
(C) It was hard for people to ride the (B) Bill was too sick to come to class.
horses for long. (C) Bill was sick of calculus class.
(D) The riders worked the horses too (D) Bill had forgotten about the
much. calculus class that morning.
3. (A) She thinks she knows when the

6
Skill 3: SUGGESTIONS AND UNCERTAINTY

Saran juga sangat umum digunakan dalam dialog singkat. Ciri dari
3
pertanyaan ini adalah terdapat kata: let’s...., why don’t, why not.
Perhatikan contoh dibawah.

Example:

On the recording, you hear:


(man) I haven’t talked with my
parents in a while (woman) Why
don’t you call them now?
(narrator) What does the woman
suggest?

In your test book, you read:


(A) Calling off his visit
(B) Talking about his parents
(C) Calling his parents in a while
(D) Phoning his family

Dalam contoh ini, ekspresi “why don’t” merupakan contoh saran, jadi saran si
wanita adalah untuk memanggil mereka. Ini berarti bahwa si wanita itu
menyarankan menelefon keluarganya sekarang (bukan nanti). Sehingga
jawaban terbaik adalah (D).

Selain itu, juga seringkali terdaoat soal terkait uncertainty atau


ketidakyakinan. Beberaoa ciri uncertainty adalah sebagai berikut: as far as
I can tell, .... isn’t it (tag), as far as I know.

Example:

On the recording, you hear:


(man) Is the exam still scheduled at 3 p.m this afternoon?
(woman) As far as I know.
(narrator) What does the woman mean?

In your test book, you read:


(A) The exam is far away.
(B) She knows that the exam schedule has been changed.
(C) She is sure that the exam is set for Thrusday.
(D) She thinks she knows when the test is

Jawaban yang tepat adalah D, karena kata think menandakan bahwa dia merasa tidak
yakin.

7
Garisbawahi ekspresi saran di setiap dialog singkat berikut. Kemudian,
pilih jawaban terbaik untuk pertanyaan itu. Ingat bahwa jawaban
terbaik adalah yang memberikan saran.

1. (man) The weather’s so beautiful 2. (woman) I have never enough


today spending money
(woman) Let’s walk to school (man) Why not get a part-time job?
instead of driving (narrator) What does the man suggest?
(narrator) What does the woman
suggest? (A) Spending less money
(B) Doing a better job at work
(A) Taking the car to school (C) Earning some money
(B) Taking a walk instead of going to (D) Spending less time at her job
school
(C) Going for a drive in the beautiful
weather
(D) Going to class on foot

TOEFL EXERCISE 3: In this exercise, listen carefully to each short


dialogue and question on the recording, and then choose the best answer
to the question. You should be particularly careful of expressions of
emphatic surprise.

1. (A) Drink the hot coffee immediately.


(B) Let the coffee cool off a bit.
(C) Serve hot coffee in a few minutes.
(D) Wait for her to serve hot coffee.
2. (A) Put ice to the drink.
(B) Boil the water first.
(C) Drink boiled water.
(D) Don’t use ice cubes.
4. (A) Take a rest.
(B) Leave earlier.
(C) Wait till ten o’clock.
(D) Take a lift.

8
Skill 4: PASSIVE

Kadang-kadang sulit untuk mengerti siapa atau apa yang melakukan tindakan dalam kalimat
4
pasif. Masalah ini sering diujikan dalam dialog singkat.

Example:

On the recording, you hear:


(man) Is that a new chair?
(woman) Yes, we just bought it last night
(Narrator) What does the woman mean?

In this test book, you read:


(A) She brought the chair with her
(B) The chair was lost for a week
(C) The chair was purchased recently
(D) She bought the last chair from the store

Dalam dialog ini, si wanita menggunakan ide aktif, “kami baru saja membelinya (kursi)”.
Jawaban yang benar untuk menggunakan gagasan pasif adalah “kursi itu dibeli”. Sehingga,
jawaban terbaik untuk pertanyaan di atas adalah (C).

Baca dialog singkat dan garisbawahi pernyataan aktif atau pasif yang
ada. Kemudian baca pertanyaan dibawah dan pilih jawaban terbaik
untuk pertanyaan itu. Pastikan Anda memahami siapa yang melakukan
tindakan dalam kalimat pasif.

1. (man) what happened to your 2. (man) When are we going to talk


notebook? about the problem with the copy
(woman) I left it in the cafeteria machine?
(narrator) What does the woman (woman) It will be discussed at the
imply? copy machine
(A) The cafeteria is to the left (narrator) What does the woman
(B) She left a note on the cafeteria door mean?
(C) She took some notes in the
cafeteria (A) She needs to make copies before
(D) The notebook was left in the the meeting
cafeteria (B) They will talk about the problem
tomorrow
(C) It is a problem to have meeting
(D) They must discuss whether or not
to have a meeting

9
TOEFL EXERCISE 4: In this exercise, match each point (A) with the
correct choices from line B.
A: Should I send the packages B: He’s already been sent
now? home.
A: I want to help. Should I B: No, that’s O.K. They’ve
sweep the porch? already been sent.
A: At least let me help you pay B: No problem. They’ve
the bills. already been paid.
A: Has June been told about B: Yes, everything has been
her operation? explained to her.
A: Is Mr. Warren ready to B: No, don’t worry. It’s
leave the hospital? already been swept.
A: I wasn’t informed. B: Mr. Warren was happy to
Someone should have told go home.
me.

Skill 5: WHO, WHAT, and WHERE (Siapa, Apa, dan Dimana) 5


Keduanya sering digunakan dalam dialog singkat dengan tujuan menarik kesimpulan. Dalam
hal ini jawabannya tidak dinyatakan dengan jelas, melainkan harus dapat disimpulkan
berdasarkan klausa yang diberikan dalam dialog. Salah satu jenis kesimpulan yang umum di
bagian tes ini adalah meminta Anda untuk menentukan siapa pembicara itu, apa aktifitas
yang dilakukan atau apa pendapat yang dikemukakan, dan dimana tempat percakapan
tersebut dilakukan berdasarkan klausa dalam dialog.

Example 1:

On the recording, you hear:


(man) What do you do during your performance?
(woman) I play the piano and sing
(narrator) Who is the woman most likely to be?
In your test book, you hear:

(A) An athlete
(B) A member of the audience
(C) A clerk in a music store
(D) A musician

Petunjuk piano dan bernyanyi dalam percakapan menggiring kita pada kesimpulan bahwa
wanita itu seorang musisi sehingga (D) adalah jawaban yang benar.

10
Example 2:

On the recording, you hear:


(man) I’d like to open an account, please
(woman) What type of account would you like, a checking or savings account?
(narrator) Where does the conversation probably take place?

In your test book, you read:


(A) In a accounting class
(B) In a bank
(C) At a service station
(D) In a market

Dalam latihan ini, baca dialog singkat dan pertanyaan dibawah,


garisbawahi petunjuk yang bisa membantu dan pilih jawaban terbaik.

1. (man) I’d like to mail this package, 2. (woman) How much are the flowers?
please (man) Ten dollars a bouquet
(woman) First or second class? (narrator) Where does this
(narrator) Who is the woman most conversation probably take place?
likely to be? (A) In a bakery
(A) A school administrator (B) In a grocery store
(B) A postal worker (C) In a florist shop
(C) A banker (D) In a garden
(D) A teacher

TOEFL EXERCISE 5: In this exercise, listen carefully to each short


dialogue and question on the recording and then choose the best answer to
the question. You will have to draw conclusions about who, what, and where.

1. (A) Although he did pass, Mark’s 4. (A) Attend a party.


exam grade wasn’t too good. (B) Go home.
(B) Mark failed his history exam. (C) Leave the woman alone.
(C) The highest grade on the (D) Disappoint his friends.
history exam went to Mark. 5. (A) He’s enjoying himself tremendously.
(D) Professor Frank didn’t pass (B) He wonders if the woman enjoyed herself.
Mark on the history exam.
(C) He wants to know what she said.
2. (A) In a department store. (D) This party hasn’t been
(B) In a stationery store.
any fun at all.
(C) At the post office.
(D) At the airport.
3. (A) She slept well after the noisy
party.
(B) She couldn’t sleep last night.
(C) She was not disturbed by
noisy party.
(D) She enjoyed the party.

11
Skill 6: AGREEMENT (Persetujuan) 6
Ungkapan persetujuan lazim digunakan dalam dialog singkat. Berikut contoh yang
menunjukkan perjanjian dengan pernyataan positif.

Example:

On the recording, you hear:


(man) I thought that the meal was overpriced
(woman) Me, too.
(narrator) What does the woman mean?

In your test book, you read:

(A) There were too many spices in the meal


(B) She has the same opinion of the meal as the man
(C) She wants to share the man‟s meal
(D) The price of the meal was great

Ungkapan “ Me, too” menunjukkan persetujuan dalam hal positif. Hal ini berarti si wanita
setuju dengan pendapat si laki-laki. Sehingga (B) adalah jawaban yang paling tepat.

Garisbawahi ekspresi yang menunjukkan persetujuan di setiap dialog


singkat. Pilih jawaban terbaik untuk pertanyaan itu. Ingat bahwa
jawaban terbaik adalah yang menunjukkan persetujuan.

1. (woman) This homework is going to 2. (woman) A trip to the park might


take forever be nice?
(man) I’ll say (man) You can say that again
(narrator) What does the man (narrator) What does the man
mean? mean?

(A) It’s going to take forever to get (A) The woman should repeat what
home she said
(B) It takes a long time to get from (B) It’s nice in the park at night
home to work (C) The woman should tell him about
(C) He and the woman have the same part of the trip
opinion about the homework (D) He agrees about the trip to the
(D) He needs to take the homework to park
class (E)

TOEFL EXERCISE 6: In this exercise, listen carefully to each short dialogue and question
on the recording and then choose the best answer to the question. Remember that the best
answer is one that shows agreement.

12
1. (A) The letter did not need more postage. home.
(B) The letter arrived last week. 4. (A) She condones what
(C) She did not put enough happened.
postage on the letter. (B) She doesn’t like what the man said.
(D) She did not put any (C) She says that she did not do
postage on the letter. it.
(D) She agrees with the
2. (A) He is hesitant about joining the trip.
man about what
(B) He wants to join his friends.
happened.
(C) He doesn’t want to join.
5. (A) This party hasn’t been any fun at all.
(D) He doesn’t understand the idea.
(B) He wonders if the
3. (A) He is going to wait for the bus.
woman enjoyed
(B) He is going to walk to his destination herself.
(C) He is going to wait for thirty minutes (C) He wants to know what he said.
only. (D) He’s enjoyed himself tremendously.
(D) He injured his legs and will return

13
1.2. LONG CONVERSATION (Percakapan Panjang)

Pembicaraan panjang seringkali berkisar tentang beberapa persoalan


terkait studi (betapa sulitnya sebuah mata kuliah, bagaimana menulis
sebuah makalah penelitian, bagaimana cara mendaftar untuk kursus,
dsb) atau tentang kehidupan secara umum (menyewa apartemen,
bermain olahraga, pergi ke bank). Percakapan juga dapat berupa
kehidupan umum di Amerika Serikat (desalinasi pasokan air, daur
ulang produk yang digunakan, kerusakan dari badai atau tipe lain dari
fenomena alam). Percakapan panjang akan diikuti beberapa buah
pertanyaan yang merujuk dari tema yang sama.

Example:

On the recording you hear:


(narrator) Question 1 through 4. Listen to a conversation between
two people who are decorating an appartment.
(woman) Hey, Walt. Do you think you could help me hang
these pictures on the wall? There are only two of them.
(man) Sure, Monica. Where do you want them to go?
(Woman) I’d like to picture of the mountains over the fireplace, and I’d like the
picture of my family over the sofa. What do you think?
(man) I think they’ll look fine there. How about if you hold the pictures while I
hammer the nails into the wall?
(woman) Okay. Let’s start with the picture of my family.

Questions:
1. On the recording you hear:
(narrator) What are the man and woman discussing?

In your test book you read: (A) Taking some pictures


(B) Hanging some plants
(C) Taking a trip to the mountains
(D) Putting some pictures on the wall

Karena si wanita meminta si pria itu untuk membantu menggantung gambar-


gambar di dinding, jawaban terbaik untuk pertanyaan ini adalah jawaban ini
(D), menarik beberapa gambar di dinding.

2. On the recording you hear:


(narrator) How many pictures are there? In your test book you read:
(A) One
(B) Two
(C) Three
(D) Four

Pertanyaan kedua bertanya berapa banyak gambar yang ada, dan si


wanita dengan jelas mengatakan bahwa ada dua, jadi jawaban terbaik
adalah jawaban (B).

14
3. On the recording you hear:
(narrator) Where is the picture of the woman‟s family going?
In your test book you read: (A) in the fireplace
(B) Above the sofa
(C) Home with Walt
(D) To the top of the mountain

Pertanyaan ketiga bertanya tentang lokasi gambar keluarga. Wanita itu


mengatakan bahwa dia akan meletakkannya di atas sofa, jadi jawaban terbaik
untuk pertanyaan ini adalah jawaban (B), di atas sofa.
4. On the recording you hear:
(narrator) What is Walt probably
going to do next? In your test book
you read: (A) Sit on the sofa
(B) Photograph Monica‟s family
(C) Hammer the nails into the walls
(D) Climb the walls

Pertanyaan terakhir menanyakan apa yang mungkin akan dilakukan oleh


Walt. Walt telah menyarankan bahwa ia harus memasang paku-paku ke
dinding, jadi jawaban terbaik adalah jawaban (C).

Questions 31 through 35 are based on the following


conversation

Woman : did you read the article that the professor assigned for
tomorrow’s class?
Man : no, not yet. Was what it about?
Woman : it was about pollution, specifically one kind of pollution called
acid rain.
Man : why is it called acid rain?
Woman : because the rain or some other kind of precipitation has been
polluted with acid
Man : where did the acid come from?
Woman : From cars of factories, anything that burns coal or oil. These are
made up mostly of sulfur dioxide and nitrogen oxides, which react
with water vapor to form sulfuric acid or nitric acid
Man : you mean that when coal or oil is burned, acid gets formed. And
when it rains or snows the acids fall back to the earth
Woman : exactly! That’s why it is so dangerous. Acid rain has been falling
over areas of northern America and northern Europe, and if this
is not checked, the effect of the air supply and plant and animal
life could be disastrous
Man : that is something important. I really need to read the article.

Questions
31. narrator: what is the topic of the conversation?

15
a. All kinds of pollution
b. How acid rain has harmed the earth
c. Pollution from cars and factories
d. The cause and possible effects of acid rain

32. Narrator: what energy sources cause acid rain?


a. nuclear power
b. electricity
c. burning coal and oil
d. solar power

1.3. LONG TALKS (Pembicaraan Panjang)

Topik pembicaraan sering tentang beberapa aspek kehidupan sekolah atau


topik dalam berita. Pembicaraan panjang bisa juga diperpendek dengan tema
perkuliahan di perguruan tinggi Amerika dan universitas luar negeri lainnya.
Dari sebuah long talks akan diberikan beberapa pertanyaan terkait
pembicaraan tersebut.

Example:

On the recording, you hear:


(narrator) Question 1 through 4. Listen to a lecture in a history class
(woman) Salt, which today seems so plentiful to us, in the past used
to be a valuable commodity. In the ancient past in
China, salt was used to make coins, and in parts of
Africa it was traded in place of money. In the Roman
Empire, soldiers were not paid in money but were
instead paid in salt. In fact, the English word “salary”
which means the amount of money that one earns,
comes from the latin root for “salt”.
Questions:

1. On the recording, you hear:


(narrator) What is the topic of the talk?
In your test book, you read: (A) Valuable commodities
(B) Salt
(C) Ancient China
(D) Money
Pertanyaan pertama menanyakan tentang topik pembicaraan. Pembicaraan dimulaidengan
topik garam dan terus berbicara tentang hal itu di seluruh bagian, jadi jawaban terbaik
adalah jawaban (B).
2. On the recording, you hear:
(narrator) What was salt used for in China?
In your test book, you read: (A) To spice food
(B) To build houses
(C) To make coins
(D) To locate Africa

16
Pertanyaan kedua menanyakan tentang penggunaan garam di Cina. Pembicara mengatakan
bahwa di Cina, Garam digunakan untuk untuk membuat koin, jadi jawaban terbaik adalah
jawaban (C).
3. On the recording, you hear:
(narrator) What does “salary” mean in English?
In your test book, you read: (A) Coins
(B) Earnings
(C) Soldiers
(D) Commodities
Pertanyaan ketiga menanyakan arti dari "gaji". Pembicara mengatakan gaji yang berarti
jumlah uang, jadi jawaban terbaik adalah jawaban (B).

4. On the recording you hear:


(narrator) What is the meaning of the root “sal” in Latin?
In your test book, you read: (A) Salt
(B) Rome
(C) Money
(D) Trade
Pertanyaan terakhir menanyakan tentang arti akar kata dari "sal". Pembicara mengatakan
bahwa "sal" berasal dari akar bahasa Latin untuk "garam, jadi jawaban terbaik adalah
jawaban (A).

17
UNIT II

STRUCTURE & WRITTEN


EXPRESSION

Skill I: BE SURE THE SENTENCE HAS A SUBJECT AND A VERB 1


Example I
was backed up for miles on the freeway.
(A) Yesterday (C) Traffic
(B) In the morning (D) Cars

In this example, the best answer is (C) because it is a singular subject that agrees with the
singular verb was.
Example II
Engineers for work on the new space program.
(A) necessary (C) hopefully
(B) are needed (D) next month

In this example you should notice immediately that the sentence has a subject (engineers),
and that there is no verb. Because answer (B), are needed, is a verb, it is the best answer.

EXERCISE 1: Underline the subjects once and the verbs twice in each of the
following sentences. Then indicate if the sentences are correct (C) or incorrect (I).
1. Last week went fishing for trout at the nearby mountain lake.
2. A schedule of the day’s events can be obtained at the front desk.
3. A job on the day shift or the night shift at the plant available.
4. The new computer program has provides a variety of helpful applications.
5. The box can be opened only with a special screwdriver.

Skill 2: BE CAREFUL OF OBJECTS OF PREPOSITIONS 2


An object of preposition is a noun or a pronoun that comes after a preposition, such as in, at,
of, to, by behind, on, and so on, to form a prepositional phrase.
(After his exams) Tom will take a trip (by boat)
This sentence contains two objects of prepositions. Exams is the object of the preposition
after and boat is the object of the preposition by.
Example
With his friend found the movie theater.
(A) has (C) later
(B) he (D) when

18
In this example you should notice the verb found and should also notice that there is no
subject. Because a subject is needed in this sentence, answer (B), he, is the best answer.
EXERCISE 2: Each of the following sentences contains one or more prepositional
phrases. Underline the subjects once and the verbs twice. Circle the prepositional
phrases that come before the verb. Then indicate if the sentences are correct (C) or
incorrect (I).
C 1. The interviews by radio broadcasters were carried live by the station.
I 2. In the last possible momen before take off took his seat in the airplane.
3. At the neighborhood flower shop, flowers in quantities of a dozen or a half dozen
can be delivered for free.
4. The progressive reading methods at this school are given credit for the improved
test scores.
5. For the last three years at various hospitals in the county has been practicing
medicine.

Skill 3: BE CAREFULL OF PRESENT PARTICIPLES 3


A Present participle is the –ing form of the verb. It is part of the verb when it is preceded by
some form of the verb be.

The man is talking to his friend.


VERB

In this sentence talking is part of the verb because it is accompanied by is.

Example
The child playing in the yard is my son.
(A) now (C) he
(B) is (D) was

In this example, you should recognize that playing is a participial adjective rather than a verb
because there is another verb in the sentence (is). In the sentence there is a complete subject
(child) and a complete verb (is), so this sentence does not need another subject or verb. The
best answer here is (A).

EXERCISE 3: Each of the following sentences contains one or more present


participles. Underline the subjects once and the verbs twice. Circle the present
participles and label them as adjectives or verbs. Then indicate if the sentences are
correct (C) or incorrect (I).
C 1. The companies offering the lowest prices will have the most customers.
ADJ.
I _ 2. Those travelers are completing their trip on Delta should report to Gate Three.
VERB
3. The artisans were demonstrating various handicrafts at booths throughout the fair.
4. The fraternities are giving the wildest parties attract the most new pledges.
5. The first team winning four games is awarded the championship.

19
Skill 4: BE CAREFULL OF PAST PARTICIPLES 4
The past participle is the form of the verb that appears with have or be. A past participle often
ends in –ed but there are also many irregular past participles.
Example
The packages mailed at the post office will arrive
Monday.
(A) have (C) them
(B) were (D) just

In this example it appears that packages is the subject and mailed is either a complete verb
or a past participle that needs a helping verb. So, answer (D) is the best answer to this
question

EXERCISE 4: Each of the following sentences contains one or more past participles.
Underline the subjects once and the verbs twice. Circle the past participles and label
them as adjectives or verbs. Then indicate if the sentences are correct (C) or incorrect
(I).
I 1. The money was offered by the client was not accepted.
VERB VERB
C 2. The car listed in the advertisement had already stalled.
ADJ B
3. The chapters were taught by the professor this morning will be on next week’s
exam.
4. The loaves of bread were baked in a brick oven at a low temperature for many
hours.
5. The ports were reached by the sailors were under the control of a foreign nation.

Skill 5: USE COORDINATE CONNECTORS CORRECTLY 5


When you have two clauses in an English sentence, you must connect the two clauses
correctly. One way to connect two clauses is to use and, but, or, so, or yet between the
clauses.
Tom is singing, and Paul is dancing.
Tom is tall, but Paul is short.
Tom must write the letter, or Paul will do it.
Tom told a joke, so Paul laughed.
Tom is tired, yet he is not going to sleep.

Example
A power failure occured, the lamps went out.
(A) then (C) later
(B) so (D) next

In this example, you should notice that the sentence needs a connector to join the two
clauses. The best answer is answer (B) because so can connect two clauses.

20
EXERCISE 5: Each of the following sentences contains more than one clause.
Underline the subjects once and the verbs twice. Circle the connectors. Then indicate
if the sentences are correct (C) or incorrect (I).
C _1. The software should be used on an IBM computer, and this computer is an IBM.
I 2. The rain clouds can be seen in the distance, but no has fallen.
3. They are trying to sell their house, it has been on the market for two months.
4. So the quality of the print was not good, I changed the toner cartridge.
_ 5. The lifeguard will warn you about the riptides, or she may require you to get out of
the water.

Skill 6: USE OTHER ADVERB CONNECTORS CORRECTLY 6


Adverb clauses can also express a number of other ideas, suh as conmtrast, condition,
manner, and place.
Study the following examples:
I will leave at 7:00 if I am ready.
In the sentence the adverb condition connector if comes in the middle of the sentence.

Example
You will get a good grade on the exam provided _ .
(A) studying (C) to study
(B) study (D) you study

In this example you should notice the adverb condition connector provided. This connector
comes in the middle of the sentence, so it must be followed by a subject and a verb. The best
answer is answer (D), which contains the subject and a verb you study.

EXERCISE 6: Each of the following sentences contains more than one clause.
Underline the subjects once and the verbs twice. Circle the connectors. Then indicate
if the sentences are correct (C) or incorrect (I).
C 1.It is impossible to enter that program if you lack experience as a teacher.
I 2.The commandant left strict orders about the passes, several soldiers left the post
anyway.
3. No one is admitted to the academy unless he or she the education requirements.
4. While most students turned the assignment in on time, a few asked for an
extension.
5. I will take you wherever need to go to complete the registration procedures.

21
TOEFL REVIEW EXERCISES (Skill 1-6):
Choose the letter of the word or group of words that best completes the sentence.
1. The three basic chords in entered its widely erratic orbit
the tonic, the around Jupiter.
dominant, and the subdominant. (A) it
(A) functional harmony (B) when
(B) functional harmony is (C) after the comet came into it
(C) functional harmony are (D) once the comet
(D) functional harmony they are 7. Each object Jupiter’s
2. Hale Telescope, at the magnetic field is deluged with
Palomar Observatory in southern electrical charges.
California, scientists can (A) enters
photograph objects severalbillion (B) it enters
light years away. (C) entering
(A) The (D) enter
(B) With the 8. As its name suggests, the Prairie
(C) They use the Wetlands Resource Center
(D) It is the the protection of wetlands on the
3. Without the proper card installed prairies of the Dakotas, Montana,
inside the computer, Minnesota, and Nebraska.
impossible to run a graphical (A) it focuses
program. (B) focuses on
(A) is definitely (C) focusing
(B) because of (D) to focus on
(C) it is 9. One of the Largest and most
(D) is powerful birds of prey in the world,
4. The charter for the Loiusiana a six-foot wingspan and
lottery was coming uo for the legs and talons roughly the size of a
renewal, spared no mans’s arms and legs.
expense in the fight to win several. (A) so the harpy has
(A) the lottery committee (B) the harpy having
(B) so the lottery committee and (C) with the harpy having
(C) so the lottery committee (D) the harpy has
(D) the lottery committee made 10. creation of such a
5. While in reality Alpha Centauri is a community was a desirable step,
triple star, to the naked the requisite political upheaval had
eye to be single star. to be accepted.
(A) it appears (A) Since the
(B) but it appears (B) The
(C) appears (C) Later, the
(D) despite it (D) It was the
6. The Sun’s gravity severely distorted
the path of the comet

22
Skill 7: USE REDUCE ADJECTIVE CLAUSE CORRECTLY 7
Adjective clause can appear in a reduced form. In the reduced form, the adjective clause
connector and the be-verb that directly follow it are omitted.
Example:
The woman who is waving to us is the tour guide.

Example
on several different television programs, the witness
gave conflicting accounts of whta had happened.
(A) He appeared(C) Appearing
(B) Who appeared (D) Appears
In this example, answer (C) is the correct answer because it is the reduced form of the clause
who appeared, and this reduced form can appear at the front of the sentence.

EXERCISE 7: Each of the following sentences contains an adjective clause, in a


complete or reduced form. Underline the adjective clauses. Then indicate if the
sentences are correct (C) or incorrect (I).
C _ 1. We will have to return the merchandise purchased yesterday at the Broadway.
I 2. The children sat in the fancy restaurant found it difficult to behave.
3. Serving a term of four eyes, the mayor of the town will face reelection next year.
4. The brand new Cadillac, purchasing less than two weeks ago, was destroyed in the
accident.
5. The fans who supporting their team always come out to the games in large
numbers.

Skill 8: USE REDUCE ADVERB CLAUSE CORRECTLY 8


Adverb clause can also appear in a reduced form. In the reduced form, the adverb connector
remains, but the subject and be-verb are omitted.
Although he is rather unwell, the speaker will take part in the seminar.
Example
When , you are free to leave.
(A) the finished report (C) the report
(B) finished with the report (D) is the report finished

The correct answer is answer (B); this answer is the reduced form of the clause when you are
finished with the report.

EXERCISE 8: Each of the following sentences contains an adjective clause, in a


complete or reduced form. Underline the adjective clauses. Then indicate if the
sentences are correct (C) or incorrect (I).
C _ 1. If not completely satisfied, you can return the product to the manufacturer.
I 2. Steve has had to learn how to cook and clean since left home.
3. The ointment can be applied where needed.
4. Tom began to look for a job after completing his master’s degree in engineering.
5. Although not selecting for the team, he attends all of the games as a fan.

23
Skill 9: INVERT THE SUBJECT AND VERB WITH QUESTION WORDS 9
• They can introduce a question, and in this case the subject and verb that follow are
inverted.
What is the homework?
• These words can join together two clauses, and in this case the subject and verb that
follwo are not inverted.
I do not know what the homework is.

Example
The lawyer asked the client why it.
(A) did he do (C) he did
(B) did he (D) did
In this example the question word why is used to connect the two clauses, so a subject and
verb are needed after this connector; this is not a question, so the subject and verb should not
be inverted. The best answer is therefore answer (C).

EXERCISE 9: Each of the following sentences contains a question word. Circle the
question words. Underline the subjects once and the verbs twice. Then indicate if the
sentences are correct (C) or incorrect (I).
I 1. The phone company is not certain when will the new directories be ready.
C _ 2. The professor does not understand why so many students did poorly on the exam.
3. How new students can get information about parking?
4. How long it has been since you arrived in the United States?
5. The jury doubts what the witness said under cross-examination.
6. What type of security does he prefer for his investments?
7. Only the pilot can tell how far the plane go on one tank of fuel.

Skill 10: INVERT THE SUBJECT AND VERB WITH PLACE EXPRESSIONS 10
• Using single words expressing place, such as here, there, or nowhere
Here is the book that you lent me.
• The subject and verb can also be inverted after prepositional phrases expressing
place.
Around the corner is Sam’s house.
Another example:
1. In the forest are many exotic birds. (the subject birds and verb are inverted because
the place expression is needed to complete the idea many exotic birds are...)
2. In the forest, I walked for many hours. (the subject I and verb walked are not inverted
because the idea I walked for many hours is complete without the place expression in
the forest)

24
Example
On the second level of the parking lot .
(A) is empty (C) some empty stalls are
(B) are empty (D) are some empty stalls

This example needs a subject and a verb to be complete. The subject and verb should be
inverted because the place expression is necessary to complete the idea some empty stalls
are...The best answer is therefore answer (D).

EXERCISE 10: Each of the following sentences contains an expression of place at


the beginning of the sentence. Circle the expressions of place. Look at the clauses
that immediately follow the place expressions and underline the subjects once and the
verbs twice. Then indicate if the sentences are correct (C) or incorrect (I).

C _ 1. In front of the house were some giant trees.


I 2. There a big house is on the corner.
3. In the cave was a vast treasure of gems and jewels.
4. At the Italian restaurant was the food too spicy for my taste.
5. In the backyard the two trees are that need to be pruned.

Skill 11: INVERT THE SUBJECT AND VERB WITH NEGATIVES 11


• When negative expressions, such as no, not or never, come at the beginning of a
sentence.
Not once did I miss a question.
• Certain words such as hardly, barely, scarcelyand only, act like negatives. If one of
these words comes at the beginning of a sentence, the subject and verb are also
inverted.
Hardly ever does he take time off.
(This means that he almost never takes time off)
• When a negative expression appears in fornt of a subject and verb in the middle of a
sentence. This happens often with the negative words neither and nor.
I do not want to go, and neither does Tom.

Example
Only in extremely dangerous situations stopped.
(A) will be the printing presses (C) that the printing presses will be
(B) the printing presses will be (D) will the printing presses be

In this example the sentence begins with the negative only, so an inverted subject and verb
are needed. Answer (D) is the best answer because it contains a correctly inverted subject
and verb, with the helping verb will, the subject printing presses, and the main verb be.

25
EXERCISE 11: Each of the following sentences contains a negative or “almost
negative” expression. Circle the negative expressions. Look at the clauses that follow
and underline the subjects once and the verbs twice. Then indicate if the sentences are
correct (C) or incorrect (I).
I 1. Never the boy wrote to his sisters.
C _ 2. On no occasion did they say that to me.
3. Steve dod not win the prize, nor did he expect to do so.
4. Did he go out of the house at no time.
5. Only once in my life gone I have to New York City.

Skill 12: INVERT THE SUBJECT AND VERB WITH CONDITIONALS 12


• When the helping verb in the conditional clause is had, should, or were, and the
conditional connector if is omitted.
If he had taken more time, the results would have been better.
Had he taken more time, the results would have been better.

Example
The report would have been accepted in checking its
accuracy.
(A) if more care (C) had taken more care
(B) more care had been taken (D) had more care been

In this example, a connector if and a subject and verb are needed, but if could be omitted and
the subject and verb inverted. Because it is correct to invert the subject more care and the
helping verb had, answer (D) is correct.

EXERCISE 12: Each of the following sentences contains a conditional with a stated
or an implied if. Circle the conditionals, or put an asterisk ( ⃰ ) where if has been
omitted. Look at the clauses that follow and underline the subjects once and the verbs
twice. Then indicate if the sentences are correct (C) or incorrect (I).
C _ 1. ⃰ Were our neighbors a bit more friendly, it would be somewhat easier to get to know
them.
I 2. There are plenty of blankets in the closet if should you get cold during the night.
3. Has he enough vacation days left this year, he will take two full weeks off in
December.
4. Had we been informed of the decision, we might have had something to say about
it .
5. I would like to know could you help me pack these boxes.

Skill 13: INVERT THE SUBJECT AND VERB WITH COMPARISONS 13


• The inversion of a subject and verb after a comparison is optional, rather tah
required, and it is a rather formal structure.
My sister spends more hours in the office than John.
My sister spends more hours in the office than John does.
My sister spends more hours in the office than does John.

26
Example
The results of the current experiment appear to be more
consistent than the results of any previous tests.
(A) them (C) they were
(B) were (D) were they

In this example, you should notice the comparison more consistent than, and you should also
understand that the results of the currect experiment is being compared with the results of
any previous tests. Because the results of any previous tests is the subject, only a verb is
needed; the best answer to this question is therefore answer (B).

EXERCISE 13: Each of the following sentences contains a comparison. Circle the
comparisons. Look at the clauses that follow and underline the subjects once and the
verbs twice. Then indicate if the sentences are correct (C) or incorrect (I).
C _1. This candidate has received more votes than has any other candidate in previous
years.
I 2. Obviously we were much more impressed with the performance than did the other
members of the audience.
3. The film that we saw last night at the festival was far better than any of the other
films.
4. The vegetables at the market this morning were far fresher than were those at the
market yesterdat .
5. I am afraid that is the condition of these tires as bad as the condition of others.

TOEFL REVIEW EXERCISES (Skill 1-13): Choose the letter of the word or group
of words that best completes the sentence.
1. variety of flowers in 4. that Emily Dickinson
the show, from simple carnationsto wrote, 24 were given titles and 7
the most exquisite roses. were published during her lifetime.
(A) A wide (A) Of the 1,800 poems
(B) There was a wide (B) There were 1,800 poems
(C) Was there (C) Because the 1,800 poems
(D) Many (D) The 1,800 poems
2. The wedges dartboard are 5. Since an immediate change was
worth from one to twenty points needed on an emergency basis,
each. by the governor to curtail
(A) they are on a railway expenditure.
(B) are on a (A) so it was proposed
(C) are they on a (B) was proposed
(D) on a (C) because of the proposal
3. producing many new movies (D) it was proposed
for release after the new season.
(A) His company is
(B) His companies
(C) The company
(D) Why the company is

27
6. In the Morgan Library in New York 9. Individual differences in brain-
City of medieval and wave activity may shed light on
renaissance manuscripts. why some people are more prone to
(A) a collection is emotional stress disorder .
(B) in a collection (A) that others are
(C) is a collection (B) and others are
(D) which is a collection (C) others are
7. Some fishing fleets might not have (D) than are others
been so ineficient in limiting their 10. squezzed, the orange
catch to target species more juice in a one-cup serving provides
strict in enforcing penalties. twice the minimum daily
(A) the government had been requirement for vitamin C.
(B) if the government had (A) It is freshly
(C) had the government been (B) It freshly
(D) if the government (C) You freshly
8. The Dewey decimal system, (D) If it freshly
currently used in the libraries
throughout the world, all
written works into ten classes
according to subject.
(A) dividing
(B) divides
(C) it would decide
(D) was divided

28
THE WRITTEN EXPRESSION QUESTIONS

Example I
The final delivery of the day is the importantest.
A B C D

If you look at the underlined words in this example you should notice immediately that
importantest is not correct. The correct superlative form of important is the most
important. Therefore, you should choose answer (D) because (D) is not correct.

Example I
The books that I read was interesting
A B C D

In this example, the singular verb was is incorrect because it does not agree with the plural
subject books; the verb should be were instead. Therefore, you should choose answer (C)
because (C) is not correct.

❖ PROBLEM WITH SUBJECT/VERB AGREEMENT

Skill 14: MAKE VERBS AGREE AFTER PREPOSITIONAL PHRASES 14


• When a prepositional phrase come between the subject and the verb, be sure that the
verb agrees with the subject.

EXERCISE 14: Each of the following sentences has one or more prepositional
phrases between the subject and verb. Circle the prepositional phrases. Underline
the subjects once and the verbs twice. Then indicate if the sentences are correct (C) or
incorrect (I).
C _1. The climbers on the sheer face of the mountain need to be rescued.
I _2. The interrogation, conducted by three police officers, have lasted for several hours.
3. The tenants in the apartment next to mine is giving a party this evening.
4. The president, surrounded by Secret Service agents, is trying to make his way to the
podium.
5. The buildings destroyed during the fire are being rebuilt at the taxpayers’ expense.

Skill 15: MAKE VERBS AGREE AFTER EXPRESSIONS OF QUANTITY 15


• When an expression of quantity is the subject, the verb agrees with the subject. In this
situation, the subject (all, most, or some) can be singular or plural, depending on
what follows the preposition of.
All (of the book) was interesting.
SINGULAR
All (of the books) were interesting.
PLURAL
All (of the information) was interesting.
UNCOUNTABLE

29
EXERCISE 15: Each of the following sentences has a quantity expression as the
subject. Underline the subjects once and the verbs twice. Circle the objects that the
objects that the verbs agree with. Then indicate if the sentences are correct (C) or
incorrect (I).
C _1. The witnesses saw that most of the fire in the hills was extingushed.
I 2. Some of the animals from the zoo was released into the animal preserve.
3. All of the students in the class taught by Professor Roberts is required to turn in
their term papers next Monday.
4. Half of the food that we are serving to the guests are still in the refrigerator.
5. We believe that some of the time of the employees is going to be devoted to quality
control.
6. All of the witnesses in the jury trial, which lasted more than two weeks, have
indicated that they believed that defendant was guilty.
7. She did not know where most of the people in the room was from.

Skill 16: MAKE VERBS AGREE AFTER CERTAIN WORDS 16


SUBJECT/VERB AGREEMENT AFTER CERTAIN WORDS
These words or expressions are grammatically singular, so they take singular verb:
anybody everybody nobody somebody each (+ noun)
anyone everyone no one someone every (+
noun)
anything everything nothing something

EXERCISE 16: Each of the following sentences contains one of the words that are
grammatically singular but have plural meanings. Underline these words once and
underline the verbs twice. Then indicate if the sentences are correct (C) or incorrect
(I).
I 1. It is impossible to believe that somebody actually admire that man.
C _ 2. Each of the doctors in the building needs to have a separate reception area.
3.The president felt that no one were better suited for the position of chief staff
advisor.
4. Everybody participating in the fund-raiser are to turn in the tickets by 8:00.
5. Because of the low number of orders, nothing has to be done now.

30
TOEFL REVIEW EXERCISES (Skill 1-16): Choose the letter of the word or group
of words that best completes the sentence.

1. several unsuccessful 4. of economics cycles


attempts, Robert Peary reached the been helpful in predicting turning
North Pole on April 6, 1909. points in cycles, they would have
(A) After been used more consistently..
(B) He made (A) Psychological theories
(C) When (B) Psychological theories have
(D) His (C) Had psychological theories
2. The musical instrument is (D) Psychologists have theories
six feet long. 5. Hospital committees
(A) is called the bass spent weeks agonizing over which
(B) it is called the bass artificial kidney candidate would
(C) called the bass receive the treatments now find
(D) calls the bass that the decision is out of their
3. One problem with all languages hands.
they are full of irregularities. (A) once
(A) when (B) that once
(B) so (C) have
(C) is that (D) once had
(D) in case

Choose the letter of the underlined word or group of words that is not correct.

6. More than half of the children in the 1,356-member district qualifies for
A B C
reduced-price of free lunches.
D
7. Five miles beyond the hills were a fire with its flames reaching up to the sky.
A B C D
8. Kettledrums, what were first played on horseback, were incorporated into the
A B C D
orchestra in the eighteenth century.

9. When is a flag hung upside down, it is an internationally recognized symbol of


A B C D
distress.
10. The museum of the Confederation in Richmond hosts an exhibition which
A
documenting the origins and history of the banner that most Americans think of as
B C D
The Confederate flag.

31
❖ PROBLEMS WITH THE FORM OF THE VERB

Skill 17: AFTER HAVE, USE THE PAST PARTICIPLE


• Verb forms after have
HAVE + PAST PARTICPLE
My friend has sung in the choir.

EXERCISE 17: Each of the following sentences contains a verb in the past or a past
participle. Underline the verbs or past participle twice. Then indicate if the sentences
are correct (C) or incorrect (I).
I 1. The young girl drunk a glass of milk.
C _ 2. Before she left, she had asked her mother for permission.
3. Having finished the tem paper, he began studying for the exam.
4. The secretary has broke her typewriter.
5. The installer should have completes the task more quickly.

Skill 18: AFTER BE, USE THE PRESENT PARTICIPLE OR THE PAST 18
PARTICPLE

VERB FORMS AFTER BE


BE + (1) present participle
(2) past participle
e.g:
We are doing our homework.
The book was taken by Tom

EXERCISE 18: Each of the following sentences contains a verb with be. Underline
the verbs twice. Then indicate if the sentences are correct (C) or incorrect (I).
I 1. At 12.00 Sam is eat his lunch.
C _ 2. We are meeting them later today.
3. The message was took by the receptionist.
4. Being hard was extremely important to him.
5. The Smiths are build their house on some property that they own in the desert.

Skill 19: AFTER WILL, WOULD, OR OTHER MODALS, USE THE BASE 19
FORM OF THE VERB

VERB FORMS AFTER MODALS


MODAL + base form of the verb
e.g:
The boat will leave at 3:00.
EXERCISE 19: Each of the following sentences contains a verb formed with a model.
Underline the verbs twice. Then indicate if the sentences are correct (C) or incorrect
(I).
C _1. The salesclerk might lower the price.
I 2. The television movie will finishes in a few minutes.
3. Should everyone arrive by 8:00?
4. The method for organizing files can be improved.
5. The machine may clicks off if it is overused.
32
TOEFL REVIEW EXERCISES (Skill 1-19): Choose the letter of the word or group
of words that best completes the sentence.
1. separates Manhattan’s (D) the job’s security
Upper East Side from the Upper 4. When on July 4, 1789,
West Side. the federal tariff, intended by the
(A) Central Park Founding Fathers to be the
(B) Where Central Park givernment’s primary source of
(C) Where is Central Park revenue, was remarkably
(D) Central Park which evenhanded.
2. Bioluminescent animals (A) was first enacted
the water or on a land. (B) first enacted
(A) live (C) was enacted first
(B) are living either (D) it first
(C) they are found in 5. inclined to push for such
(D) can be found in a reduction, it would probably not
3. The purpose of a labor union is to be succesful.
improve the working conditions, (A) The Office of Management
, and pay of its members. (B) The Office of Management was
(A) jobs are secure (C) In the Office of Management
(B) to be secure (D) Where the Office of
(C) job security Management

Choose the letter of the underlined word or group of words that is not correct.

6. Helium has the most low boiling point of all substances.


A B C D
7. There is twenty-six-bones in the human foot, fourteen of them in the toes.
A B C D
8. Extension of the countdown hold to fourteen hours was order to give crews
A B
more time to repair wiring and clear away equipment.
C D
9. The study demonstrates that neither experience or awarness will improve chances
A B C D
of success.
10. Some of the eye movements used in reading is actually unnecessary.
A B C D

33
Skill 20: RECOGNIZE ACTIVE AND PASSIVE MEANINGS 20
ACTIVE The subject does the action of the verb.
PASSIVE The subject receives the action of the verb.
e.g:
Active sentence: We mailed the package at the post office.
Pasive sentence: The letter was mailed by us today before noon.

EXERCISE 20: Each of the following sentences has a passive meaning. Underline
the twice the verbs that should be passive. Then indicate if the sentences are correct
(C) or incorrect (I).
I 1. The car parked in a no-parking zone.
C _2. The physics exam began just a few minutes ago.
3. Everything to organize the picnic has already done.
4. The police investigated him because of his unusual actions.
5. The package containing the necessary samples has just sent.

TOEFL REVIEW EXERCISES (Skill 1-20): Choose the letter of the word or
group of words that best completes the sentence.
1. Big Dipper; a seven- (C) which is located
star constellation in the Shape of a (D) whose location is
cup, is part of Ursa Major. 3. impressive chapter in the
(A) The problem had already book was the chapter on
(B) The problem is already Stuart’scientific theories.
(C) The problem had already been (A) It was the most
(D) The problem has already (B) The most
2. The Military Academy at West (C) Most
Point on the west bank of (D) Most of the
the Hudson River, north of New
York City.
(A) located
(B) is located

Choose the letter of the underlined word or group of words that is not correct.

4. The first fish have appeared on the earth approximately 500 million years ago.
A B C D
5. Only rarely sound waves are of a single frequency encountered in practice.
A B C D
6. Cameos can be carved not only from onyx and sardonyx or from agate.
A B C D
7. Although most of the wild horses in the western range have already been rounded
A B
up, the most remote the area, the greater the possibility that wild horses can still
C D
be found.
8. During this period, $206 was spend annually on food by families in the lower
A B C D

34
third income bracket.

9. The dangers of noise are, unfortunately, not as clear-cut than are those from
A B C
most other health hazards.
D
10. In a recent survey of Americans, more than 75 percent expressed the view that the
A B
government it should take a more active role in health care.
C D

Skill 21: USE –ED AND –ING ADJECTIVES CORRECTLY 21


• Verb forms ending in –ed and –ing can be used as adjectives.
e.g:
The woman cleans the car.
VERB

The cleaning woman worked on the car.


ADJECTIVE

The woman put the cleaned car back in the garage.


ADJECTIVE

EXERCISE 21: Each of the following sentences contains either an –ed or an –ing
verbal adjective. Circle the verbal adjectives. Draw arrows to the words they describe.
Then indicate if the sentences are correct (C) or incorrect (I).
I 1. The teacher gave a quiz on the just completing less on.
C _2. There is a fascinating movie at the theater tonight.
3. They thought that it had been a very satisfied dinner.
4. The empty bottles are to the left, and the filling bottles are to the right.
5. For lunch at the restaurant she ordered a mixed salad.

Skill 22: USE ARTICLES WITH SINGULAR NOUNS 22


• A singular noun must have an article (a, an, the) or some other determiner such as
my or each. (A plural noun or uincountable noun may or may not have an article.)
e.g:
I have money. (uncountable – no article needed)
I have books. (countable plural – no article needed)
I have a book. (countable singular - article needed)

EXERCISE 22: The following sentences contain different types of nouns. Circle only
the countable singular nouns. Mark where articles (or determiners) have been
omitted. Then indicate if the sentences are correct (C) or incorrect (I).
I 1. She is taking trip with friends.
C _2. In my yard there are flowers, trees, and grass.
35
3. The manager sent memo to his employees.
4. There is car in front of the building.
5. The child and his friends are having milk and cookies.

TOEFL REVIEW EXERCISES (Skill 1-22): Choose the letter of the word or
group of words that best completes the sentence.
1. In economics, “diminishing (C) it is
returns” describes (D) more
resource inputs and production. 3. Rarely remove the entire
(A) among root of a dandelion because of its
(B) when it is length and sturdiness.
(C) among them (A) can the casual gardener
(D) the relationship between (B) the casual gardener
2. When lava reaches the surface, its (C) the casual gardener will
temperature can be ten times (D) does the casual gardener’s
boiling water.
(A) the temperature
(B) that of

Choose the letter of the underlined word or group of words that is not correct.

4. Operas can be broadly classified as either comedies or they are tragedies.


A B C D
5. Tungsten has the highest melting point of all metals, and for this reason it is often
A
use in equipment that must withstand high temperatures.
B C D
6. Whereas there are forty-three ant species in Great Britain, the same amount of ant
A B C
species can be found in a single tree in Peru.
D
7. People voice theirs opinions first in small groups or among friends and
A B C
acquaintances.
D
8. Inside the Lincoln Memorial is a large statue of Lincoln make from white marble.
A B C D
9. Detailed photometric data of the area just north of Triton’s equatorial region
A
indicate the existence of a thin, transparent layers of frost.
B C D
10. U.S. census figures indicate that people with only an elementary education can
A B
earn just half as much as college graduations.
C D

36
UNIT III

READING
COMPREHENSION
Skill 1: ANSWER MAIN IDEA
QUESTIONS CORRECTLY
1
Almost every reading passage on the TOEFL
test will have a question about the main
idea of a passage. You will be asked to
identify the topic, subject, title, primary
idea, or main idea.
If a passage consists only one paragraph,
you should study the beginning of that paragraph to determine the main idea.

Example I
The passage:
In the philosophy of John Dewey, a sharp distinction is made between
“intelligence” and “reasoning.” According to Dewey, intelligence is the only absolute
way to achieve a balance between realism and idealism, between practically and wisdom
of life. Intelligence involves “interacting with other things and knowing them,” while
reasoning is merely the act of an observer, “... a mind that beholds or grasps object
outside the world of things...”
With reasoning, a level of mental certainty can be achieved, but it is through intelligence
that control is taken of events that shape one’s life.
The question:
What is the topic of this passage?
(A) The intelligence of John Dewey
(B) Distinctions made by John Dewey
(C) Dewey’s ideas on the ability to reason
(D) How intelligence differs from reasoning in Dewey’s works

The best answer is therefore (D); the idea of how intelligence differs from reasoning comes
from the first sentence of the passage, which mentions a sharp distinction ... between
“intelligence” and “reasoning.”
If a passage consists of more than one paragraph, you should study at the beginning of each
paragraph to determine the main idea.
MAIN IDEA QUESTIONS
HOW TO IDENTIFY THE QUESTION What is the topic of the passage?
What is the subject of the passage?
What is the main idea of the passage?
What is the author’s main point in the
passage?
With what is the author primarily
concerned?
Which of the following would be the best title?

37
WHERE TO FIND THE ANSWER The answer to this type of question can
generally be determined by looking at the first
sentence of each paragraph.
HOW TO ANSWER THE QUESTION 1. Read the first line of each paragraph.
2. Look for a common theme or idea in the
first lines.
3. Pass your eyes quickly over the rest of the
passage to check that you really have found
the topic sentence(s).
4. Eliminate any definitely wrong answers
and choose the best answer from the
remaining choices.

TOEFL EXERCISE

Fort Knox, Kentucky, is the site of a U.S. army post, but it is even more renowned for
the Fort Knox Bullion Depository, the massive vault that contains the bulk of the U.S.
government’s gold deposits. Completed in 1936, the vault is housed in a two-story building
constructed of granite, steel, and concrete; the vault itself is made of steel and concrete and
has a door that weighs more than twenty tons. Naturally, the most up-to-date security
devices available are in place at Fort Knox, and the army post nearby provides further
protection.
1. Which of the following best describes 2. Which of the following would be the
the topic of the passage? best title for this passage?
(A) The city of Fort Knox, Kentucky (A) The Massive Concrete Vault
(B) The federal gold depository (B) Fort Knox Security
(C) The U.S. army post at Fort Knox (C) Where the U.S. Keeps Its Gold
(D) Gold bullion (D) A Visit to Kentucky

One identifying characteristic of minerals is their relative hardness, which can be


determined by scratching one mineral with another. In this type of test, a harder mineral can
scratch a softer one, but a softer mineral is unable to scratch the harder one. The Mohs’ scale
hardness scale is used to rank minerals according hardness. Ten minerals are listed in this
scale, ranging from talc with a hardness of 1 to diamond with a hardness of 10. On this scale,
quartz (number 7) is harder than feldspar (number 6) and is therefore able to scratch it;
however feldspar is unable to make a mark on quartz.

3. Which of the following best states the 4. Which of the following would be the
subject of this passage? best title for this passage?
(A) The hardness of diamonds (A) the hardness of a mineral can be
(B) Identifying minerals by means of a determined by its ability to make a
scratch test. mark on other minerals
(C) Feldspar on the Mohs’ scale (B) diamonds, with a hardness of 10 on
(D) Recognizing minerals in their the Mohs’ scale, can scratch all
natural state other minerals
(C) a softer mineral cannot be
scratched by a harder mineral
(D) talc is the first mineral listed on the
Mohs’ scale
38
Skill 2: ANSWER STATED DETAIL QUESTIONS CORRECTLY 2
A stated detail question asks about one piece of information in the passage rather than the
passage as a whole.

Example
The passage:
Williamsburg is a historic city in Virginia situated on a peninsula between two
rivers, the York and the James. It was settled by English colonists in 1633, twenty-six
years after the first permanent English colony in America was settled at Jamestown. In the
beginning the colony at Williamsburg was named Middle Plantation because of its location
in the middle of the peninsula. The site for Williamsburg had been selected by the
colonists because the soil drainage was better three than at the Jamestown location, and
there were fewer mosquitoes.
The questions:
1. According to the passage, Williamsburg is located
(A) on an island
(B) in the middle of a river
(C) where the York and James meet
(D) on a piece of land with rivers on two sides

• The best answer to the first question is answer (D); with rivers in two sides is closest
in meaning to between two rivers.
STATED DETAIL QUESTIONS
HOW TO IDENTIFY THE QUESTION According to the passage, ...
It is stated in the passage...
The passage indicates that...
The author mentions that...
Which of the following is true...?
WHERE TO FIND THE ANSWER The answers to these questions are found in
order in the passage.
How to answer the question 1. Choose a key word in the question.
2. Skim in the appropriate part of the
passage for the key word or idea.
3. Read the sentence that contains the key
word or idea carefully.
4. Look for the answer that restates an idea
in the passage.
5. Eliminate the definitely wrong answers
and choose the best answer from the
remaining choices.

39
TOEFL EXERCISE

Ice ages, those periods when ice covered extensive areas of the Earth, are known to
have occured at least six times. Past ice ages can be recognized from rock strata that show
evidence of foreign materials deposited by moving walls of ice or melting glaciers. Ice ages
can also be recognized from land formations that have been produced from moving walls of
ice, such as U-shaped valleys, sculptured landscapes, and polished rock faces.

1. According to the passage, what 2. The passage covers how many different
happens during an ice age? methods of recognizing past ice ages?
(A) Rock strata are recognized by (A) One
geologists. (B) Two
(B) Evidence of foreign materials is (C) Three
found. (D) Four
(C) Ice covers a large portion of the 3. According to the passage, what in the
Earth’s surface. rock strata is a clue to geologists of a
(D) Ice melts six times. past ice age?
(A) Ice
(B) Melting glaciers
(C) U-shaped valleys

3
(D) Substances from other areas

Skill 3: FIND PRONOUN REFERENTS

• Whenever you are asked which noun a pronoun refers to, you should look before the
pronoun to find the noun.
Example
The passage:
Carnovorous plants, such as the sundew and the Venus-flytrap, are generally found in
humid areas where there is an inadequate supply of nitrogen in the soil. In order to survive, these
plants have developed mechanism to trap insects within their foliage. They have digestive fluids
to obtain the necessary nitrogen from the insects. These plants trap the insects in a variety ways.
The sundew has sticky hairs on its leaves; when an insect lands on these leaves, it gets caught up
in the sticky hairs, and the leaf wraps itse.lf around the insect. The leaves of the Venus-flytrap
function more a like a trap, snapping suddenly and forcefully shut around an insect
The questions:
1. The pronoun “they” in line 4 refers to
(A) humid areas
(B) these plants
(C) insects
(D) digestive fluids

In the first example, you should understand from the context that these plants have digestive
fluids to obtain the necessary nitrogen from the insects, so the best answer to this question is
answer (B).
PRONOUN REFERENTS
HOW TO IDENTIFY THE QUESTION The pronoun “...” in line X refers to which
of the following?
WHERE TO FIND THE ANSWER The line where the pronoun is located is
generally given in the question. The noun
that the pronoun refers to is generally found
before the pronoun.
40
HOW TO ANSWER THE QUESTION 1. Find the pronoun in the passage. (The
line where the pronoun can be found is
generally stated in the question.)
2. Look for nouns that come before the
pronoun.
3. Read the part of the passage before the
pronoun carefully.
4. Eliminate any definitely wrong answers
and choose the best answer from the
remaining choices.

TOEFL EXERCISE
The full moon that occurs nearest the equinox of the Sun has become known as the
harvest moon. It is a bright moon which allows farmers to work late into the night for several
nights; they can work when the moon is at its brightest to bring in the fall harvest. The
harvest moon of course occurs at different times of the year in the northern and southern
hemisphere, the harvest moon occurs in September at the time of the autumnal equinox. In
the southern hemisphere, the harvest moon occurs in March at the time of the vernal
equinox.
1. The pronoun “It” in line 2 refers to 2. The pronoun “they” in line 3 refers to
(A) the equinox (A) farmers
(B) the Sun (B) nights
(C) the harvest moon (C) times of the year
(D) the night (D) northern and southern
hemispheres

Skill 4: ANSWER IMPLIED DETAIL QUESTIONS CORRECTLY 4


You will sometimes be asked to answer a question by drawing a conclusion from a specific
detail or details. Questions of this type contain words implied, inferred, likely, or probably.
Example
The passage:
The Hawaiian language is a melodious language in which all words are derived from an
alphabet of only twelve letters, the five vowels a, e, i, o, u, and the seven consonants h, k, l, m, n,
p, w. Each syllable in the language ends in a vowel, and two consonants never appear together, so
vowels have a much higher frequency in the Hawaiian language than they do in English.
This musical-sounding language can be heard regularly by visitors to the islands. Most
Hawaiians speak English, but it is quite common to hear English that is liberally spiced with
words and expressions from the traditional language of the culture. A visitor may be greeted with
the expression aloha and may be referred to as a malihini because he is a newcomer to the island.
This visitor may attend an outside luau where everyone eats too much and may be invited
afterwards to dance the hula.
The questions:
1. Which of the following is probably NOT a Hawaiian word?
(A) mahalo (C) meklea
(B) mahimahi (D) moana
2. It is implied that a luau is
(A) a dance (C) a concert
(B) a feast (D) a language

41
In this example, you should see the part of the passage where it states that in the Hawaiian
language two consonants never appear together. From this you can draw the conclusion
that answer (C), maklea, is probably not a Hawaiian word because the consonants k and l
appear together in this word, so answer (C) is the best answer to this question.

IMPLIED DETAIL QUESTIONS


HOW TO IDENTIFY THE QUESTION It is implied in the passage that...
It can be inferred from the passage that...
It is most likely that...
What probably happened ...?
WHERE TO FIND THE ANSWER The answers to these questions are found in
order in the passage.
HOW TO ANSWER THE QUESTION 1. Choose a key word in the question.
2. Scan the passage for the key word (or a
related idea).
3. Carefully read the sentence that contains
the key word.
4. Look for an answer that could be true,
according to that sentence.

TOEFL EXERCISE

Two types of trees from the same family of trees share honors in certain respects as
the most impressive of trees. Both evergreen conifers, the California redwood (Sequoia
sempervirens) and the giant sequoia (Sequoiandendron giganteum) are found growing
natively only in the state of California. The California redwood is found along the northern
coast of the state, while the giant sequoia is found inland and at higher elevations, along the
western slopes of the Sierra Nevadas.
The California redwood is the tallest living tree and is in fact the tallest living thing on
the face of the Earth; the height of the tallest redwood on record is 385 feet (120 meters).
Though not quite as tall as the California redwood, with a height of 320 feet (100 meters), the
giant sequoia is nonetheless the largest and most massive of living things; giant sequoias
have been measured at more than 100 feet (30 meters) around the base, with weights of more
than 6,000 tons.

1. It is implied in the passage that (B) a coastal community


(A) The leaves of only the California (C) a group of lakes
redwood turn brown in the autumn (D) a mountain range
(B) The leaves of only the giant sequoia
turn brown in the winter 3. Which of the following is implied in the
(C) The leaves of both types of trees in passage?
the passage turn brown in the (A) The giant sequoia is taller than the
winter California redwood.
(D) The leaves of neither type of tree in (B) The California redwood is not as
the passage turn brown in the big around as the giant sequoia.
winter (C) The California redwood weighs
2. It can be inferred from the passage that more than the giant sequoia.
the Sierra Nevada are (D) Other living things are larger than
(A) a type of giant redwood the giant sequoia
42
Skill 5: ANSWER TRANSITION QUESTIONS CORRECTLY 5
The topic of the preceding or following paragraph is not directly stated, and you must draw a
conclusion to determine what is probably in these paragraphs.

Example
The passage:
Another myth of the oceans concerns Davy Jones, who in folklore is the mean-spirited
sovereign of the ocean’s depths. The name “Jones” is thought by some etymologists to have been
derived from the name “Jonah,” the Hebrew prophet who spent three days in a whale’s belly.
According to tradition, any object that goes overboard and sinks to the bottom of the ocean
is said to have gone to Davy Jones’s locker, the ocean-sized, mythical receptable for anything that
falls into the water. Needless to say, any sailor on the seas is not so eager to take a tour of Davy
Jones’s locker, although it might be a rather interesting rip considering all the treasures located
there.
The questions:
1. The paragraph preceding this passage most probably discusses
(A) the youth of Davy Jones
(B) Davy Jones’s career as a sailor
(C) a different traditional story from the sea
(D) preparing to travel on the ocean
2. The topic of the paragraph following the passage most likely is
(A) Valuable items located at the bottom of the ocean
(B) Where Davy Jones is found today
(C) Jonah and the whale
(D) Preventing objects from falling overboard

The first question asks about the topic of the preceding paragraph, so you must look
at the beginning of the passage and draw a conclusion about what probably came before.
Since the passage begins with the expression another myth of the oceans, you should
understand that the new passage is going to present a second myth of the oceans and the
previous passage probably presented the first myth of the oceans. A myth is traditional story,
so the best answer to this question is answer (C).

The second question asks about drawing a conclusion about what probably comes
after. The passage ends with the mention of all the treasures located there, and there is in
Davy Jones’s locker, or at the bottom of the ocean; this is probably going to be the topic of the
next paragraph. The best answer to the second question is therefore answer (A).

TRANSITION QUESTIONS
The paragraph preceding the passage
probably...
HOW TO IDENTIFY
What is most likely in the paragraph following
the passage?
The answer can generally be found in the first line
of the passage for a preceding question. The
WHERE TO FIND THE ANSWER
answer can generally be found in the last line for a
following question.
1. Read the first line for a preceding question.
2. Read the last line for a following question.
HOW TO ANSWER THE 3. Draw a conclusion about what comes before or
QUESTION after.
4. Choose the answer that is reflected in the first
or last line of the passage.

43
TOEFL EXERCISE
Another program instrumental in the popularization of science was Cosmos. This series,
broadcast on public television, dealt with topics and issues from varied fields of science. The
principal writer and narrator of the program was Carl Sagan, a noted astronomer and
Pulitzer Prize-winning author.

1. The paragraph preceding this passage 2. The paragraph following this passage
most probably discusses most likely contains information on
(A) a different scientific television what?
series (A) The popularity of science
(B) Carl Sagan’s scientific (B) The program Cosmos
achievements (C) The astronomer Carl Sagan
(C) the Pulitzer Prize won by Carl (D) Topics and issues from various
Sagan fields of science
(D) public television

When a strong earthquake occurs on the ocean floor rather than on land, a tremendous
force is exerted on the seawater and one or more large, destructive waves called tsunamis can
be formed. Tsunamis are commonly called tidal waves in the United States, but this is really
an inappropriate name in that the cause of the tsunami is an underground earthquake rather
than the ocean’s tides.
Far from land, a tsunami can move through the wide open vastness of the ocean’s tides.
600 miles (900 kilometers) per hour and often can travel tremendous distances without
losing height and strength. When a tsunami reaches shallow coastal water, it can reach a
height of 100 feet (30 meters) or more and can cause tremendous flooding and damage to
coastal areas.

3. The paragraph preceding the passage 4. Which of the following is most likely
most probably discusses the topic of the paragraph following
(A) Tsunamis in various parts of the the passage?
world (A) The cause of tsunamis
(B) The negative effects of tsunamis (B) The destructive effects of
(C) Land-based earthquakes tsunamis on the coast
(D) The effect of tides on tsunamis (C) The differences covered by
tsunamis
(D) Topics and issues from various
fields of science

Skill 6: FIND DEFINITIONS FROM STRUCTURAL CLUES 6


When you are asked to determine the meaning of a word, it is possible
1) That the passage provides information about the meaning of the word
2) That there are structural clues to tell you that the definition of a word is included in the
passage.

44
Example

The passage:
One of the leading schools of psychological thought in the twentieth century
was behaviorism – the belief that the role of the psychologist is to study behavior,
which is observable, rather than conscious or unconscious thought, which is not.
Probably the bestknown proponent of behaviorism is B.F. Skinner, who was famous
for his research on how rewards and punishments influence behavior. He came to
believe that positive reinforcements such as praise, food, or money were more
effective in promoting good behavior than negative reinforcement, or punishment.

The questions:
1. What is “positive reinforcement” 2. What is “negative reinforcement” in
in line 6? line 7?
(A) A gift (A) A promotion
(B) A reward (B) A reward
(C) A bribe (C) A surprise
(D) A penalty (D) A punishment

The best answer for the first question is answer (B), while the best answer for the second
question is answer (D).

STRUCTURAL CLUES
HOW TO IDENTIFY THE QUESTION What is ...?
What is the meaning of...?
What is true about...?
TYPES OF CLUES Punctuation: comma, parentheses, dashes
Restatement: or, that is, in other words, i.e.
Example: such as, for example, e.g.
WHERE TO FIND THE ANSWER Information to help you determine what
something means will generally be found
after the punctuation clue, the restatement
clue, or the example clue.
HOW TO ANSWER THE QUESTION 1. Find the word in the passage.
2. Locate any structural clues.
3. Read the part of the passage after the
structural clue carefully.
4. Eliminate any definitely wrong answers
and choose the best answer from the
remaining choices.

TOEFL EXERCISE

The teddy bear is a child’s toy, a nice, soft stuffed animal suitable for cudding. It is,
however, a toy with an interesting history behind it.
Theodore Roosevelt, or Teddy as he was commonly called, was president of the
United States from 1901 to 1909. He was unusually active man with varied pastimes, one of
which was hunting. One day the president was invited to take part in a bear hunt; and
inasmuch as Teddy was president, his hosts wanted to ensure that he caught a bear. A bear
was captured, clunked over the head to knock it out, and tied to a tree; however, Teddy, who
45
really wanted to actually hunt, refused to shoot the bear and in fact demanded that the bear
be extricated from the ropes; that is, he demanded that the bear be set free.
The incident attracted a lot of attention among journalists. First a cartoon-drawn by
Clifford K. Berryman to make fun of this situation-appeared in the Washington Post, and the
cartoon was widely distributed and reprinted throughout the country. Then toy
manufactures began producing a toy bear which they called a “teddy bear.” The teddy bear
became the most widely recognized symbol of Roosevelt’s presidency.

1. According to the line 1 of the passage, 3. The word “extricated” in line 9 is


what is a “teddy bear”? closest in meaning to which of the
(A) A ferocious animal following?
(B) The president of the United States (A) Released
(C) A famous hunter (B) Tied up
(D) A plaything (C) Hunted
2. In line 4, “pastimes” could best be (D) Shot
replaced by 4. In line 10, a “cartoon” could best be
(A) things that occurred in the past described as
(B) previous jobs (A) a newspaper
(C) hunting trips (B) a type of teddy bear
(D) leisure activities (C) a drawing with a message
(D) a newspaper

Skill 7: DETERMINE WHERE SPECIFIC INFORMATION IS FOUND 7


Sometimes the final question in a reading passage will ask you to determine where in the
passage a piece of information is found. The best way to approach this type of question is to
study the question to determine the information that you are looking for and then to go to the
lines listed in the answers and skim for that information.

Example
Meteor Crater; a great crater approximately 40 miles east of Flagstaff, Arizona, is generally thought
by scientists to have formed as a result of the impact of a 60,000-ton meteor about 50,000 years ago. The
meteor, made of nickel and iron, disintegrated on impact and spread half a billion tons of rock over the
surface of the land. The masiveness of the meteor can only be imagined from the mammoth size of the
crater, which measures a mile in diameter and three miles around the top. The rim of the crater rises more
than ten miles on a clear day.
The questions:
1. Where in the passage does the author discuss the composition of the meteor?
(A) Lines 1-4
(B) Lines 4-5
(C) Lines 6-8
(D) Lines 8-10
2. Where in the passage does the author mention the distance from which the crater can be
seen?
(A) Lines 1-4
(B) Lines 4-5
(C) Lines 6-8
(D) Lines 9-10

The best answer to the first question is answer (B). While the best answer to the second
question is answer (D).

46
QUESTIONS ABOUT WHERE IN THE PASSAGE
HOW TO IDENTIFY THE QUESTION Where in the passage. . . ?
WHERE TO FIND THE ANSWER The answer can be in any of the lines listed in
the answers to the question.
HOW TO ANSWER THE QUESTION 1. Choose a key word or idea in the
question.
2. Skim the lines in the passage that are
listed in the answers to the question. You
should skim for the key word or idea.
3. Choose the answer that contains the line
numbers of a restatement ofthe question.

TOEFL EXERCISE

Beavers generally live in family clusters consisting of six to ten members. One cluster
would probably consist of two adults, one male and one female, and four to eight young
beavers, or kits. A female beaver gives birth each spring to two to four babies at a time.
These baby beavers live with their parents until they are two years old. In the springtime of
their second year they are forced out of the family group to make room for the new babies.
These two-year-old beavers then proceed to start new family clusters of their own.

1. Where in the passage does the author 3. Where in the passage does the author
give the name of a baby beaver? state the age at which beavers must go
(A) Line 1 out on their own?
(B) Line 2 (A) Line 1
(C) Line 3 (B) Line 2
(D) Lines 4-5 (C) Line 3
2. Where in the passage does the author (D) Lines 4-5
mention the time of year when new 4. Where in the passage does the author
baby beavers are born? indicate why the young beavers must
(A) Line 1 leave their parents’ home?
(B) Line 2 (A) Line 1
(C) Line 3 (B) Line 2
(D) Lines 4-5 (C) Line 3
(D) Lines 4-5

PASSAGE TWO (Questions 5-7)


Chamber music received its name because it was originally intended to be performed
in small rooms in private homes rather than huge concert halls or theaters. Today it has
evolved into small ensemble music in which each performer in the ensemble plays an
individual part.
The compositions written for this type of performance can easily be classified into
three distinct periods, each with its style of music and instrumentation. In the earliest period
(1450-1650), the viol and other instrumental families developed considerably, and
instrumental music took its first steps toward equal footing with vocal music. In the second
period (1650-1750), trio sonatas dominated. These ensemble compositions were often
written for two violins and a cello; the harpsichord was also featured in various compositions
of this period. In the modern period (after 1750), the preponderance of chamber music was
written for the string quartet, an ensemble composed of two violins, a viola, and a cello.
47
5. Where in the passage does the author 7. Where in the passage does the author
discuss the modern definition of mention music written for four
chamber music? strings?
(A) Lines 2-3 (A) Lines 2-3
(B) Lines 4-5 (B) Lines 4-5
(C) Lines 8-9 (C) Lines 7-9
(D) Lines 9-11 (D) Lines 9-11
6. Where in the passage does the author
mention the time of year when new
baby beavers are born?
(A) Lines 2-3
(B) Lines 4-5
(C) Lines 7-9
(D) Lines 9-11

Skill 8: DETERMINE THE TONE, PURPOSE, OR COURSE 8


Example
Military awards have long been considered symbolic of royalty, and thus when the United States
was a young nation just finished with revolution and eager to distance itself from anything tasting of
monarchy, there was strong sentiment against military decoration. For a century, from the end of the
Revolutionary War until the Civil War, the United States awarded no military honors. The institution of
the Medal of Honor in 1861 was a source of great discussion and concern. From the Civil War until
World War I, the Medal of Honor was the only military award given by the United States government,
and today it is awarded only in the most extreme cases of heroism. Although the United States is still
somewhat wary of granting military awards, several awards have been instituted since World War I.
The questions:
1. The tone of the passage is
(A) angered
(B) humorous
(C) outraged
(D) informational
2. The author’s purpose in this passage is to
(A) describe the history of military awards from the Revolutionary War to the Civil War
(B) demonstrate an effect of America’s attitude toward royalty
(C) give an opinion of military awards
(D) outline various historical symbols of royalty
3. The passage would probably be assigned reading in a course on
(A) general science
(B) psychology
(C) American history
(D) interior decoration

1. Answer (D)
2. Answer (B)
3. Answer (C)

48
TONE, PURPOSE, OR COURSE
HOW TO IDENTIFY TONE: What is the tone of the passage?
THE QUESTION PURPOSE: What is the author’s purpose in this passage?
COURSE: In which course would this reading be
assigned?
WHERE TO FIND TONE: There will be clues throughout the passage that
THE ANSWER the author is showing some emotion rather than
just presenting facts.
PURPOSE: Draw a conclusion about the purpose from the
main idea and supporting details.
COURSE: Draw a conclusion about the course from the
topic of the passage and the supporting details.

HOW TO ANSWER TONE: 1. Skim the passage looking for clues that the
THE QUESTION author is showing some emotion.
2. Choose the answer that identifies the emotion.
PURPOSE: 1. Study the main idea in the topic sentence and
the details used to support the main idea.
2. Draw a conclusion about the purpose.
COURSE: 1. Study the main idea in the topic sentence and
the details used to support the main idea.
2. draw a conclusion about the course.

TOEFL EXERCISE

Truman Capote’s In Cold Blood (1966) is a well-known example of the “nonfiction


novel,” a popular type of writing based upon factual events in which the author attempts to
describe the underlying forces, thoughts, and emotions that lead to actual events. In Capote’s
book, the author describes the sadistic murder of a family on a Kansas farm, often showing
the point of view of the killers. To research the book, Capote interviewed the murderers, and
he maintains that his book presents a faithful reconstruction of the incident.

1. The purpose of this passage is to 3. This passage would probably be


(A) discuss an example of a particular assigned reading in which of the
literary genre following courses?
(B) tell the story of In Cold Blood (A) Criminal Law
(C) explain Truman Capote’s reasons (B) American History
for writing In Cold Blood (C) Modern American Novels
(D) describe how Truman Capote (D) Literary Research
researched his nonfiction novel.
2. Which of the following best describes
the tone of the passage?
(A) Cold
(B) Sadistic
(C) Emotional
(D) Descriptive

49
READING REVIEW

PASSAGE ONE (Questions 1-6)


Another noteworthy trend in twentieth-century music in the U.S. was the use of folk
and popular music as a base for more serious compositions. The motivation for these
borrowings from traditional music might be a desire on the part of a composer to return to
simpler forms, to enhance patriotic feelings, or to establish an immediate rapport with an
audience. For whatever reason, composers such as Charles Ives and Aaron Copland offered
compositions featuring novel musical forms flavored with refrains from traditional
Americana. Ives employed the whole gamut of patriotic songs, hymns, jazz, and popular
songs in his compositions, while Copland drew upon folk music, particularly as sources for
the music he wrote for the ballets Billy the Kid, Rodeo, and Appalachian Spring.

1. The paragraph preceding this passage (C) why certain composers borrowed
most probably discusses from folk and popular music
(A) nineteenth-century music (D) if Copland really featured new
(B) one development in music in the musical forms
twentieth century 4. Which of the following is not listed in
(C) the works of Aaron Copland the passage as a source for Ive’s
(D) the history of folk and popular compositions?
music (A) National music
2. Which of the following best describes (B) Religious music
the main idea of the passage? (C) Jazz
(A) Traditional music flavored some (D) American novels
American musical compositions in 5. Where in the passage does the author
the last century. list examples of title of Copland’s
(B) Ives and Copland used folk and works?
popular music in their (A) Lines 1-2
compositions. (B) Lines 2-4
(C) A variety of explanations exist as to (C) Lines 4-6
why a composer might use (D) Lines 6-9
traditional sources of music. 6. The passage would most probably be
(D) Traditional music is composed of assigned reading in which of the
various types of folk and popular following courses?
music. (A) American History
3. It can be inferred from this passage (B) The History of Jazz
that the author is not sure (C) American Music
(A) when Ives wrote his compositions (D) Composition
(B) that Ives and Copland actually
borrowed from traditional music

50
PASSAGE TWO (Questions 7-13)
The rattlesnake has a reputation as a dangerous and deadly snake with a fierce hatred
for humanity. Although the rattlesnake is needed a venomous snake capable of killing a
human, its nature has perhaps been somewhat exaggerated in myth and folklore.
The rattlesnake is not inherently aggressive and generally strikes only when it has
been put on the defensive. In its defensive posture the rattlesnakes raises the front part of its
body off the ground and assumes an S-shaped form in preparation for a lunge forward. At
the end of a forward thrust, the rattlesnake pushes its fangs into the victim, threby injecting
its venom.
There are more than 30 species of rattlesnakes, varying in length from 20 inches to 6
feet and also varying in toxicity of venom. In the United States there are only a few deaths
annually from rattlesnake, with a mortality rate of less than 2 percent of those attacked.

7. Which of the following would be the (C) is partially off the ground
best title for this passage? (D) assumes it s prepared by thrusting
(A) The Exaggerated Reputation of the its fangs into the ground
Rattlesnake 11. It can be inferred from the passage that
(B) The Dangerous and Deadly (E) all rattlesnake bites are fatal
Rattlesnake (F) all rattlesnake bites are not equally
(C) The Venomous Killer of Humans harmful
(D) Myth and Folklore about Killers (G) the few deaths from rattlesnakes
8. According to the passage, which of the bites are from six-foot snakes
following is true about rattlesnakes? (H) deaths from rattlesnake bites have
(A) They are always ready to attack. been steadily increasing
(B) They are always dangerous and 12. The word “mortality” in line 10 is
deadly. closest in meaning to
(C) Their fierce nature has been (A) percentage
underplayed in myth and folklore (B) illness
(D) Their poison can kill people. (C) death
9. The word “posture” in line 5 is closest (D) survival
in meaning to which of the following? 13. The author’s purpose in this passage is
(A) Mood to
(B) Fight (A) warn readers about the extreme
(C) Position danger from rattlesnakes
(D) Strike (B) explain a misconception about
10. When a rattlesnake is ready to defend rattlesnakes
itself, it (C) describe a rattlesnake attack
(A) lies in an S-shape on the ground (D) clarify how rattlesnakes kill human
(B) lunges with the back part of its
body

51
PASSAGE THREE (Questions 14-21)

For a century before the Erie Canal was built, there was much discussion among the
general population of the Northeast as to the need for connecting the waterways of the Great
Lakes with the Atlantic Ocean. A project of such monumental proportions was not going to
be undertaken and completed without a supreme amount of effort.
The man who was instrumental in accomplishing the feat that was the Erie Canal was
DeWitt Clinton. As early as 1812, he was in the nation’s capital petitioning the federal
government or financial assistance on the project, emphasizing what a boon to the economy
of the country the canal would be; his efforts with the federal government, however, were not
successful.
In 1816, Clinton asked the New York State Legislature for the funding for the canal,
and this time he did succeed. A canal commission was instituted, and Clinton himself was
made head of it. One year lates, Clinton was elected governor of the state, and soon after,
contruction of the canal was started.
The canal took eight years to complete, and Clinton was on the first barge to travel the
length of the canal, the Seneca Chief, which departed from buffalo on October 26, 1825, and
arrived in New York City on November 4. Because of the success of the Erie Canal, numerous
other canals were built in other parts of the country.

14. The information in the passage 18. In what year did the actual building of
(A) gives a cause followed by an effect the canal get underway?
(B) is in chronological oder (A) In 1812
(C) lists opposing viewpoints of a (B) In 1816
problem (C) In 1817
(D) is organized spatially (D) In 1825
15. When did Clinton ask the U.S. 19. The Seneca Chief was
government for funds for the canal? (A) The name of the canal
(A) One hundred years before the canal (B) The name of a boat
was built (C) Clinton’s nickname
(B) In 1812 (D) The nickname of buffalo
(C) In 1816 20. Where in the passage does the author
(D) In 1825 mention a committee that worked to
16. The word “boon” in line 7 is closest in develop the canal?
meaning to which of the following? (A) The first paragraph
(A) Detriment (B) The second paragraph
(B) Disadvantage (C) The third paragraph
(C) Benefit (D) The fourth paragraph
(D) Cost 21. The paragraph following the passage
17. The pronoun “it” in line 12 refers to most probably discusses
which of the following? (A) The century before the bulding of
(A) The New York State Legislature the Erie Canal
(B) The canal (B) Canals in different U.S. locations
(C) The commission (C) The effect of the Erie Canal on the
(D) The state governor Buffalo area
(D) Clinton’s career as governor of
New York

52
PASSAGE FOUR (Questions 22-33)
The Celtic languages are a group of languages of northern Europe that are
descendents of the Indo-European family of languages. These languages developed from the
language of the Celts, a warlike civilization originating in the eastern part of central Europe,
in the northern Alps, and along the Danube during that Bronze Age. The Celts reached the
height of their civilization during the Iron Age, the last five centuries B.C., and then fanned
out from their original homeland into many parts of continental Europe and across the
channel and into the British Isles. Celtic languages were spoken in much of western Europe
during Pre-Roman and Roman times. Place names of Celtic origin can be found today all
over the British Isles and France, in northern Spain and Italy, and in Switzerland and parts of
Germany.
Rather than one language, the Celtic languages consist of two distinct clusters: the
Gaelic group and the Brythonic group. These two clusters of languages most likely developed
from dialects of the same language, the language of the Celts in their original homeland.
These two dialects were most likely mutually intelligible to some degree as late as the fourth
century. The Gaelic group of Celtic languages consists of Irish, Scottish, and Manx, the
language of the Isle of Man. The Brythonic group of Celtic languages includes Welsh,
Cornish, Breton, and Gaulish, the language of Gaul prior to the days of the Roman Empire,
with its Latin-speaking population.
Many, though not all, of the Celtic languages are either extinct or are in the process of
becoming extinct. Gaulish apparently disappeared around 600 A.D. Cornish and Manx both
actually became extinct, the former in the nineteeth century and the latter just a few decades
ago, but both are being revived and are now taught in a few schools each. Scottish, Irish, and
Breton are all declining in use. There are under a hundred thousand speakers of Scottish
Gaelic, mostly on the northern Hebridean Islands; there are more than a hundred thousand
speakers of Irish, mainly in the western counties of Ireland; there are about a half million
speakers who use Breton on a daily basis. In all these situations, though, the rate of
transmission to new generations is low, and this does not bode well for the survival of these
languages. Of all the Celtic languages, perhaps only Welsh has a strong hold on the future.
22. The author’s purpose in the passage is to (D) at the height of their civilization
(A) describe the past and present of a 1,500 years ago
related set of languages 25. The expression “fanned out” in line 5
(B) list the major characteristics of Celtic could best be replaced by
languages (A) spread out
(C) outline the major achievements of the (B) called off
Celts (C) got lost
(D) explain how languages manage to (D) turned out
survive without changing 26. The Brythonic group of languages does
23. According to the passage, the Celtic NOT include
languages did NOT (A) Welsh
(A) develop from the Indo-European (B) Cornish
language family (C) Manx
(B) originate in the British Isles (D) Breton
(C) exist before the time of the Roman 27. It is implied in the passage that Gaulish
Empire (A) first surfaced after the Roman
(D) provide any Italian place names Empire
24. The passage states that the Celts were (B) has been revived in the last century
(A) peaceful farmers (C) is declining in use
(B) unheard of during the Bronze Age (D) was replaced by Latin
(C) at their peak during the Iron Age
53
28. The main idea of the third paragraph is 31. This passage would most likely be
that assigned reading in a course on
(A) all Celtic languages are extinct (A) archeology
(B) a few Celtic languages disappeared (B) European literature
(C) some Celtic languages are flourishing (C) historical linguistics
(D) most Celtic languages are either dead (D) Bronze Age civilizations
or dying 32. Where in the passage does the author
29. It is NOT true according to the passage explain when the two clusters of Celtic
that both Cornish and Manx languages were still understood by
(A) were once considered extinct members of each group of speakers?
(B) became extinct in the same century (A) Lines 2-4
(C) are being resuscitated (B) Lines 6-7
(D) may be taught in some academic (C) Lines 10-11
institutions (D) Lines 12-13
30. According to the passage, the percentage 33. The paragraph following the passage most
of young people learning Scottish, Irish, likely discusses
and Breton is (A) how Welsh is surviving
(A) nonexistent (B) efforts to classify Celtic languages
(C) languages that preceded Celtic languages
(B) not high
in Europe
(C) increasing
(D) the causes of language extinction
(D) quite robust

54

Anda mungkin juga menyukai